How to find the volume of a sphere [spherical coordinates]

Click For Summary
The discussion focuses on finding the volume of a hemisphere using spherical coordinates, with participants questioning the accuracy of calculations and the use of limits. There are concerns about potential mistakes in the mathematical steps, particularly regarding subtraction versus multiplication and the correctness of cosine values. Participants also express frustration over an incomplete integral step and the ambiguity of terms like "a" and "r" in related threads. Additionally, there is a reminder to adhere to forum etiquette by not opening multiple threads on the same topic and maintaining the homework template for clarity. The thread concludes with a note that it has been closed due to these issues.
Another
Messages
104
Reaction score
5
27848640_1605179276230025_786094297_n.jpg
i don't know to using limit of r ?
 

Attachments

  • 27848640_1605179276230025_786094297_n.jpg
    27848640_1605179276230025_786094297_n.jpg
    10.5 KB · Views: 809
Physics news on Phys.org
i'm sorry i mean hemi-sphere of : sqrt[x^2+y^2+(z-a)^2] = R
 
I think there are mistakes between line 3 and 4. Where did you get the subtraction from? Shouldn't it be a multiplication? Also the cosine values seem wrong. And finally, why did you stop at the easiest of all integrals at the end ##\int r^2dr## from ##r= 0## to ?
 
Please do not open more than one thread with the same topic, especially if the two are both ambiguous: with or without cone, what is ##a## needed for and what is ##r## in your other thread. Furthermore, do not delete the homework template, use it! It makes reading a lot easier and if you delete it, it can be viewed as disrespectful to those who are willing to answer.

Thread closed.
The duplicate is https://www.physicsforums.com/threads/how-to-find-hemi-sphere-on-the-cone.939376/#post-5938849
 
Question: A clock's minute hand has length 4 and its hour hand has length 3. What is the distance between the tips at the moment when it is increasing most rapidly?(Putnam Exam Question) Answer: Making assumption that both the hands moves at constant angular velocities, the answer is ## \sqrt{7} .## But don't you think this assumption is somewhat doubtful and wrong?

Similar threads

  • · Replies 7 ·
Replies
7
Views
1K
Replies
3
Views
2K
  • · Replies 2 ·
Replies
2
Views
3K
  • · Replies 3 ·
Replies
3
Views
3K
Replies
6
Views
1K
  • · Replies 2 ·
Replies
2
Views
2K
  • · Replies 9 ·
Replies
9
Views
2K
  • · Replies 6 ·
Replies
6
Views
3K
  • · Replies 15 ·
Replies
15
Views
3K
  • · Replies 9 ·
Replies
9
Views
9K